Ammissione Scuola Normale 1977 - quesito 5

Sdílet
Vložit
  • čas přidán 23. 08. 2024
  • Dimostrare la NON-divisibilità: due metodi a confronto

Komentáře • 259

  • @fiorenzo.a
    @fiorenzo.a Před rokem +128

    n^2+1 = n^2-1 + 2 = (n+1) (n-1) + 2
    Essendo consecutivi, uno solo tra n-1, n e n+1 è multiplo di 3.
    Per i 2 casi diversi da n il prodotto è chiaramente multiplo di 3 ma sommando il 2 non resta multiplo; se invece n è multiplo di 3 chiaramente n^2 lo è ancora ma sommando 1 non resta multiplo.

    • @GaetanoDiCaprio
      @GaetanoDiCaprio  Před rokem +29

      Assolutamente corretta, grazie! Il tuo commento merita di essere in primo piano 👏

    • @Mario-Betti
      @Mario-Betti Před rokem

      Ciao... Scusate, non ho capito bene... Quest'eguaglianza l'ho capita:
      n^2+1 = n^2-1 + 2 = (n+1) (n-1) + 2
      Ma da qui in avanti...? Me lo rispiegate?

    • @fiorenzo.a
      @fiorenzo.a Před rokem +8

      @@Mario-Betti probabilmente sono stato un po' veloce e stringato nei vari passaggi.
      Provo a spiegare meglio:
      Prendiamo n-1, n e n+1: essendo 3 interi consecutivi uno (uno solo) di loro è multiplo di 3.
      Vediamo quindi cosa succede a n^2+1 quando n mod 3 è sia 0, 1 e 2.
      Caso a) n-1 multiplo di 3 cioè (n-1) mod 3 = 0 quindi n mod 3 = 1;
      Caso b) n multiplo di 3 cioè n mod = 0;
      Caso c) n+1 multiplo di 3 cioè n mod 3 = 2.
      Data l'uguaglianza ricavata prima è evidente che nei casi a) e c) si ottiene che dividendo per 3 il numero n^2+1 da resto 2, infatti (n-1)*(n+1) è multiplo di 3.
      Resta il caso b) che banalmente da resto 1 essendo n^2 multiplo di 3.
      Spero adesso sia più chiaro.

    • @Mario-Betti
      @Mario-Betti Před rokem

      @@fiorenzo.a Perfetto, ora è chiarissimo. Grazie.

    • @carlocartello
      @carlocartello Před rokem +8

      Semplice ed elegante. Le due dimostrazioni del video in realtà sono la stessa dimostrazione

  • @sus2349
    @sus2349 Před rokem +2

    Grazie professor Di Caprio! Io mi sono riscritta all'università a 35 anni, quindi dopo tanti anni da quando avevo iniziato. E di conseguenza moltissime cose non le ricordavo più. E i tuoi video mi stanno aiutando molto, perché spieghi le cose in modo chiaro e preciso, riuscendo anche a renderle più semplici. Invece molti professori a volte fanno il contrario, sicuramente senza volerlo, ma riescono a rendere le cose ancora più complicate di quello che sono, facendoti passare la voglia di provare a comprenderle, perché pensi di non riuscirci. La matematica è così bella e affascinante, ma indubbiamente complicata, e avere la capacità di spiegarla in modo semplice, trasmettendone anche gli aspetti più interessanti, è un dono che non tutti gli insegnanti hanno. Quindi davvero complimenti. E grazie!

  • @lulipoldbloom
    @lulipoldbloom Před rokem +17

    la seconda dimostrazione è elegantissima e non c'avrei mai pensato. Grazie mille per i video

  • @FilippoBuracchi
    @FilippoBuracchi Před rokem +18

    Una volta me la cavavo discretamente a matematica (grazie al mio prof delle medie che riuscì a darmi la forma mentis necessaria, non smetterò mai di ringraziarlo abbastanza) ma poi col tempo e il lavoro che non ne richiede chissà quanta, me la sto dimenticando. Questi brevi video sono ottimi per riportare a galla il sommerso e imparare qualcosa di nuovo! Grazie!

    • @GaetanoDiCaprio
      @GaetanoDiCaprio  Před rokem

      Grazie!

    • @danielesommo
      @danielesommo Před rokem +1

      ​@@GaetanoDiCaprioAndando alle medie e sentendo spesso nominare il quadrato di binomio, ho dimostrato che n^2+1 non fosse divisibile per tre pensando proprio al q. Di binomio, infatti un numero n si può scrivere come 3x(il numero di n in cui tre entra in n)+y (il numero restante non divisibile per 3)
      (3x+y)^2+1=9x^2+ 3xy+y^2+1
      Essendo 9x^2 e 3xy divisibili per tre, si possono anche non considerare
      Y potrebbe essere solo 2 numeri-> 1 e 2
      2^2+1=4+1=5 non divisibile
      1^2+1=1+1=2 non divisibile

    • @GaetanoDiCaprio
      @GaetanoDiCaprio  Před rokem +1

      @@danielesommo Sì ma con due piccole correzioni: 6xy non 3xy e poi y può anche essere 0

    • @danielesommo
      @danielesommo Před rokem

      @@GaetanoDiCaprio sì, grazie mille

    • @IvanFromItalia
      @IvanFromItalia Před 11 měsíci

      ​@@danielesommose sei uno studente delle medie ed hai trovato questa soluzione da solo allora sei bravissimo !! 😮

  • @user-nc1ts8nj9d
    @user-nc1ts8nj9d Před rokem +1

    Sono molto contento di aver trovato questo canale

  • @rosariov.2760
    @rosariov.2760 Před rokem +29

    questo quesito è equivalente a dimostrare che n^2=2 (mod 3) non ha soluzione; infatti 0×0=0 (mod 3), 1×1=1 (mod 3) e 2×2=1 (mod 3)

  • @enricocalcagno5401
    @enricocalcagno5401 Před rokem +1

    io l'ho risolto graficamente sul foglio a quadretti: un numero al quadrato è rappresentabile come un quadrato di lato n quadretti. anche scegliendo un numero di quadretti che permetta di dividere il quadrato in 3 parti uguali, c'è sempre quel quadretto in più che non fa venire un numero intero.
    Grazie per i bei video e le spiegazioni!

    • @GaetanoDiCaprio
      @GaetanoDiCaprio  Před rokem

      Con i quadretti ti ci vorrebbero infiniti fogli per dimostrarlo completamente 😉. Grazie per il commento positivo!

    • @giuserastelli4245
      @giuserastelli4245 Před rokem

      @@GaetanoDiCaprio mi piace la pratica dei quadretti (val più la pratica della grammatica); metodo empirico anche se per la dimostrazione davvero ci vorrebbe un tempo infinito ...

  • @guidoantonelli5549
    @guidoantonelli5549 Před rokem +2

    Per il teorema (non ultimo) di Fermat poiché 3 è primo si ha:
    n^2 (mod 3) = 1 se 3 non divide n
    n^2 (mod 3) = 0. se 3 divide n
    Sommando 1 si ottiene rispettivamente 2 ed 1 è quindi si conclude che l'affermazione è vera.

    • @GaetanoDiCaprio
      @GaetanoDiCaprio  Před rokem +1

      Direi perfetta! Complimenti

    • @guidoantonelli5549
      @guidoantonelli5549 Před rokem +2

      @@GaetanoDiCapriograzie per l'apprezzamento. Aggiungerei solamente che il problema è generalizzabile nel modo seguente.
      Per ogni primo p dispari ed un qualsiasi intero n l'espressione:
      n^(p-1) + m + kp (k intero qualsiasi)
      non è divisibile per p se:
      1

    • @GaetanoDiCaprio
      @GaetanoDiCaprio  Před rokem +1

      @@guidoantonelli5549 Fantastico

  • @mariaurgu8686
    @mariaurgu8686 Před rokem +1

    Mi piacerebbe che ci fossero piùriferimenti reali..grazie!

    • @GaetanoDiCaprio
      @GaetanoDiCaprio  Před rokem +1

      In che senso?

    • @mariaurgu8686
      @mariaurgu8686 Před rokem

      Nel senso che sorvola di sana pianta molte logiche sottese alla dimostrazione

    • @GaetanoDiCaprio
      @GaetanoDiCaprio  Před rokem

      Farò tesoro del suo commento per i prossimi video

    • @mariaurgu8686
      @mariaurgu8686 Před rokem

      @@GaetanoDiCaprio sarò molto interessata al suo prossimo video..grazie !

  • @vittorioemanueleferrante7107

    Per induzione, il passo induttivo sta nel dimostrare che è vero per n+1, quando è vero per n. Quindi che è vero per (n+1)^2 + 1. Ma questo è uguale a n^2 + 1 + 2n +1. Dato che è vero per n, allora la divisione di n^2+1 lascia un resto di 1 o di 2, che va sommato a 2n+1. Nel primo caso ne sorte un numero pari, che non è divisibile per 3; nel secondo, ne sorte un numero pari più 3, che ovviamente non è divibile per 3.

    • @GaetanoDiCaprio
      @GaetanoDiCaprio  Před rokem

      Errato, mi dispiace. Un numero pari può benissimo essere divisibile per 3, ad esempio 6, 12, ecc.

  • @mariapiapiro3718
    @mariapiapiro3718 Před rokem

    Grazie

  • @andy85s71
    @andy85s71 Před rokem

    Ho fatto la stessa dimostrazione, incredibile!!!
    P.s.Errata corrige al minuto 03:17 è stato digitato per errore 6 anziché 4.

  • @francescogaliandro662
    @francescogaliandro662 Před rokem +2

    Si poteva sfruttare il piccolo teorema di Fermat.
    Se 3 divide n^2+1, sicuramente n non è un multiplo di 3 altrimenti 3 dividerebbe 1 (ciò si ottiene sfruttando le proprietà della divisibilità) e questo è assurdo.
    Quindi 3 non divide n, quindi per il piccolo teorema di Fermat n^2 è congruo a 1 modulo 3, quindi n^2+1 è congruo a 2 modulo 3, che non è congruo a 0 modulo 3. Pertanto n^2+1 non è divisibile per tre per ogni n in Z si potrebbe anche dire, poiché ciò vale considerando un intero a piacere.

  • @otaku_gamerita
    @otaku_gamerita Před rokem +1

    Bel video grazie

  • @federicogaetani4610
    @federicogaetani4610 Před rokem +1

    Dimostrazione stile ubriacone:
    -------
    Per n multiplo di 3
    n^2 è multiplo di 3
    n^2 +1 non lo è perché la somma delle sue cifre non è divisibile per 3
    ------
    Per n non multiplo di 3 se dimostriamo che (n^2 -1)/3 è sempre intero (figata!) otteniamo che l'espressione (n^2 -1)/3 + 2/3 = (n^2 +1)/3 non è mai intera come richiesto dal quesito. Questo è facilmente verificabile per gli n non multipli di 3 che possono essere del tipo 3m+1 e 3m-1

  • @gimopirozzi2469
    @gimopirozzi2469 Před rokem

    molto carino. Grazie

  • @18feb2013
    @18feb2013 Před rokem

    Belle diostrazioni! Non so se la seguente sia già stata proposta nei commenti:
    N = 3i con i intero positivo
    Dobbiamo dimostrare se è vero che
    3i = n^2 + 1 => i = (n^2 + 1)/3
    =>
    i = (n^2)/3 +1/3 (1)
    Distinguiamo 2 casi:
    Se n è divisibile per 3, quindi n = 3k con k intero positivo, al secondo membro di (1) il primo termine è sicuramente intero; quindi rimane 1/3 (resto di 1), e l'uguaglianza non è mai soddisfatta.
    Se n non è divisibile per 3, abbiamo 2 sottocasi: o il precedente n -1 lo è; oppure il successivo n+1.
    Primo sottocaso: equivale a dire che n-1 = 3k => n = 3k+1
    n^2 = 9(k^2)+6k+1
    Secondo sottocaso: equivale a dire che n+1 = 3k => n = 3k-1
    n^2 = 9(k^2)-6k+1
    In entrambi i sottocasi, sostituendo in (1), si ha
    i = [9(k^2) +/- 6k+2]/3
    Poiché i primi due termini del secondo membro risultano sempre interi, rimangono 2/3 (resto di 2), e l'uguaglianza non è mai soddisfatta.

  • @math_art64
    @math_art64 Před rokem +1

    Mi scusi, ho provato a dividere n^2+1 per 3, e ottengo n^2/3+1/3, a questo punto, poiché il quoziente deve essere un numero intero vuol dire che n^2 /3 deve essere pari a 2/3 o a una frazione equivalente(che sommata a 1/3 dia l'intero) e quindi ottengo che n^2/3=2/3 semplifico i 3 e ottengo che n^2=2. E quindi n=√2 che è va contro le condizioni iniziali. Crede che possa essere una dimostrazione valida?

    • @GaetanoDiCaprio
      @GaetanoDiCaprio  Před rokem

      No, purtroppo no. Se cerchi ci sono almeno due proposte di soluzione simile alla tua nei commenti, dove spiego perché questo approccio è errato. Grazie comunque per la condivisione!

  • @verianoveracini3182
    @verianoveracini3182 Před rokem +13

    Non mi chiedete perché l'algoritmo mi ha proposto questo video, ma ... la seconda dimostrazione è identica alla prima. Ovvero è stato utilizzato non un metodo diverso, ma solo un linguaggio più forbito. Una dimostrazione diversa presuppone una strada diversa.

    • @GaetanoDiCaprio
      @GaetanoDiCaprio  Před rokem +3

      In parte sono d'accordo, infatti le due dimostrazioni sono equivalenti, ma "Identiche" è un'altra cosa... Grazie del commento e porta pazienza con l'algoritmo, sbaglia spesso

    • @Ssfgs
      @Ssfgs Před rokem

      Uuuh ma che bravo bimbo che conosci le parole difficili come algoritmo. Da grande puoi fare lo scienziatoooo

    • @idk-bi8dw
      @idk-bi8dw Před rokem

      @@Ssfgs beh non so tu ma a sforzarsi un attimo a pensare (da queloo che hai scritto neanchrle credo tu sia in grado di metterti a pensare ragionevolmente) se una parola esiste sul dizionario è perché va conosciuta e usata, non so che collegamento ci sia con lo scienzisto e il conoscere le parole ma da uno che perde tempo su youtube a scrivere certe cazzate non potevo aspettarmi molto

  • @ivannapolitano2202
    @ivannapolitano2202 Před rokem +2

    Bellissimo ♡. Ho solo due piccole note da fare. (1) La soluzione così presentata va benissimo in Z ma se siamo in N allora o prendiamo il k in Z oppure dovremmo testare a mano anche i numeri 1 e 2 perché "3k", "3k+1" e "3k+2" non possono rappresentarli se k è intero positivo. (2) Nel secondo caso, volendo, si può prendere "-1" come rappresentante della classe di "2" così da unificare i due casi.
    Comunque complimenti ♡.

    • @GaetanoDiCaprio
      @GaetanoDiCaprio  Před rokem +7

      Grazie per l'osservazione. In realtà l'asserto in questo modo è stato dimostrato per tutti i numeri interi, quindi vale in particolare per gli interi positivi. Non c'è bisogno di aggiustamenti. Grazie comunque per l'osservazione!

    • @gianfrancoimperiale9698
      @gianfrancoimperiale9698 Před rokem +1

      Vale per tutti i numeri interi , segui bene !
      Ciao

  • @voltydequa845
    @voltydequa845 Před rokem

    Al netto di ridondanza in fatto di coefficienti vari (3k):
    iniziale: n^2 + 1
    premessa: k mod 3 = 0
    a) k^2 + 1 => 1
    b) (k + 1)^2 + 1 => k^2 + 2k + 1 + 1 => 2
    c) (k - 1)^2 + 1 => k^2 - 2k + 1 + 1 => 2
    c-bis (k + 2)^2 + 1 => k^2 + 4k + 4 + 1 => 5 => 2
    ----
    Il metodo della congruenza non è altro che una sostituzione del n con (n mod 3), quindi si può applicare la precedente, ovvero (k + mod), senza temere che qualcosa ci potrebbe essere sfuggito.

    • @GaetanoDiCaprio
      @GaetanoDiCaprio  Před rokem

      Grazie del commento ma onestamente non ho capito bene il senso, ossia ha guardato il video?

    • @voltydequa845
      @voltydequa845 Před rokem

      @@GaetanoDiCaprio Si. Il problemino l'avevo risolto senza vedere il video (la "papa pronta" non fa per me 😀), e poi ho avuto la pazienza di vedere per intero anche il video. Avevo notato quei 3k che avevo giudicato come ridondanza. Se facciamo ipotesi che un numero è divisibile per 3 non ci serve un moltiplicatore per 3. Non so se lo sa, ma nell'ambito dei problemi di scacchi non è permessa la ridondanza - ogni pezzo deve avere una sua funzione, anche se passiva. Tradotto vuol dire che non deve essere possibile togliere un pezzo. Ma lì il dominio è ben definito perché assai limitato (semplice), mentre nella matematica le strade sono più infinite di quelle di Dio.
      Quindi prenda quel mio come pedanteria di uno ossessionato con la essenza - ché lei lo merita, dato che la generosità è da pochi, e l'interesse per la matematica di ancor meno. :)

    • @GaetanoDiCaprio
      @GaetanoDiCaprio  Před rokem +1

      No, continuo a non capire quale sarebbe la "ridondanza". Non so giocare a scacchi, per cui mi inchino a ciò che mi riferisce sugli stessi, per quanto riguarda la matematica, invece, e nella fattispecie la mia dimostrazione le garantisco che non c'è nulla di ridondante

    • @voltydequa845
      @voltydequa845 Před rokem

      @@GaetanoDiCaprio Se i tre tipi di numeri possono essere suddivisi come multipli di 3 + mod, ne consegue che possono essere suddivisi come divisibili per tre + mod. O, detta in altri termini - se partiamo dal presupposto che i numeri, del dominio mod 3, si possono suddividere in 3k + mod ({0, 1, 2}), allora 3 risulta superfluo in quanto fungente da premessa della suddivisione. Non ho bisogno del moltiplicatore 3 dato che tutte e tre le ipotesi (gruppi di numeri) si basano sulla divisibilità per tre. La consideri, qualora meritevole, come una idea sul come si potrebbe fare applicando, in un modo diverso, la seconda dimostrazione.
      Perfettamente possibile che ridondante possa essere la mia percezione (soggettiva) di ridondanza. Cestini se lo percepisce come una questione di lana caprina. Cordiali saluti!

  • @aleottoloi8782
    @aleottoloi8782 Před rokem +2

    n²+1, divisibile per 3 ?
    IMPONGO (n²+1)/3 = N ( intero )
    n² / 3 + 1/3 = N
    =》 n²/3 dovrebbe essere uguale ad un multiplo di 2/3
    il chè non e' possibile, perchè n² sempre diverso da 2.

    • @giulial
      @giulial Před rokem

      Infatti n = radq di 2 , vuol dire n irrazionale, contro le ipotesi. Proprio quello che ho scritto 7 ore fa, ma non è stato accolto

    • @GaetanoDiCaprio
      @GaetanoDiCaprio  Před rokem

      Cosa vuol dire che n^2/3 dovrebbe essere uguale ad un multiplo di 2/3? Il concetto di multiplo ha senso soltanto per i numeri interi. Non ve la prendete, purtroppo la colpa è della scuola italiana che non insegna proprio nulla sugli interi. L'unica cosa che puoi "concludere" con il tuo ragionamento è che n^2/3 dovrebbe essere uguale a N-1/3. E allora?

  • @ValerioPattaro
    @ValerioPattaro Před rokem

    In modulo 3 si ha:
    0^2=0, 1^2=1 e 2^2=1.
    Siccome non viene mai due direi che è vero.

  • @eliocopter
    @eliocopter Před rokem

    Un altro metodo sarebbe stato usare il principio di induzione, che dice che un ipotesi basata su un numero n è confermata solo se sostituendo a n lo 0 e sostituendola in un successivo passaggio con n+1 l'ipotesi viene confermata in entrambi i casi. Infatti qui avremmo che il primo passaggio è 0+1 che non è divisibile per tre e quindi si puo già dire che l'ipotesi non è confermata

    • @GaetanoDiCaprio
      @GaetanoDiCaprio  Před rokem +2

      Credo che il principio di induzione non ti sia molto chiaro. Se vuoi usare quel metodo in questo problema, la proprietà da dimostrare è "n^2+1 non è divisibile per 3". Quindi dimostrarla per il caso base non basta affatto! Devi dimostrarla anche nel passo induttivo, come tutte le dimostrazioni per induzione.

  • @riccardorizzi79
    @riccardorizzi79 Před rokem

    👍👍👍

  • @mathechne
    @mathechne Před 10 měsíci

    benissimo anche con classi di congruenza 0,1,2

  • @dinochiari3647
    @dinochiari3647 Před rokem +3

    Invece n²-1 può essere divisibile per 3 se n² non è multiplo di 9 voglio dire se n non è divisibile per 3. Anche n³+1 può essere divisibile per 3 con n=2 quindi 2³+1=8+1=9. Invece n³-1 è divisibile per 3 con n=4 quindi 4³-1=64-1=63.

    • @GaetanoDiCaprio
      @GaetanoDiCaprio  Před rokem +4

      Interessante. In realtà si può dire di più: n^2-1 è SEMPRE divisibile per 3 se n non è divisibile per 3.

  • @stefania6337
    @stefania6337 Před rokem

    Spiegazioni perfetta...ma ho letto che qualcuno aveva dato la risposta esatta ricordando una lezione fatta dal suo professore alle medie
    Oggi però non mi pare che le lezioni alle medie siano così approfondite o sbaglio? Grazie

    • @GaetanoDiCaprio
      @GaetanoDiCaprio  Před rokem

      Non lo so, non insegno alle medie ma al liceo. Quello che so è che a qualsiasi livello scolare si possono incontrare (se si è fortunati) professori eccellenti.

  • @alessiodaini7907
    @alessiodaini7907 Před rokem

    vedo che qualcuno ha avuto la stessa idea mia di usare la dimostrazione con mod(3). Bene così

  • @gerardonastro6391
    @gerardonastro6391 Před rokem +1

    Bella dimostrazione

  • @davidbaroncini9976
    @davidbaroncini9976 Před rokem

    salve, per la preparazione a questi quesiti di matematica non mi sembra ci sia mai uno schema risolutorio generale, ma mi sembra molto quasi una forma d'arte? è così?

    • @GaetanoDiCaprio
      @GaetanoDiCaprio  Před rokem +1

      Purtroppo la realtà (secondo me) è proprio l'opposto: questi quesiti sembrano "arte" solo perché la maggior parte della matematica scolastica viene insegnata male: ci si convince che la matematica è un insieme di procedimenti standard per problemi standard. Quest'ultima è la vera distorsione (grottesca) della materia, mentre l'utilizzo combinato di tecniche, concetti e creatività costituisce la vera natura della matematica.

  • @dudupls
    @dudupls Před rokem

    Ciao, io ho pensato alle funzioni y=x^2 + 1 e y=3x. Studiando le due intersezioni, si nota che (3+-sqrt(5))/2 non sono soluzioni intere. Sarebbe una dimostrazione accettabile?

    • @GaetanoDiCaprio
      @GaetanoDiCaprio  Před rokem

      No, è errato. La quantità x^2+1 non deve essere uguale a 3x ma a 3z con z intero.

  • @albertocerrone3518
    @albertocerrone3518 Před rokem

    domanda off topic, in rete girano queste equazioni molto stpide, per alcuni forse scritte in modo ambiguo ma per me non lo so, per come mi è stata insegnata la matematica anche all'università(dal professor Antonio Corbo Esposito) 8/2(2+2), chi dice 16, chi dice 1. ma la matematica non è un opinione ed io non vedo ambiguità. 8/2(2+2) equivale a scrivere 8/2*(2+2) il che mi porta ad avere 16 come risultato.(prima risolvo quello che è dentro la parentesi per poterla eliminare e poi da sinistra a destra svolgo le operazioni visto che hanno la stessa priorità)

    • @GaetanoDiCaprio
      @GaetanoDiCaprio  Před rokem

      Preferirei commenti inerenti il video, grazie (p.s. sì, la sua interpretazione è corretta)

  • @DonatoGreco
    @DonatoGreco Před rokem

    Belle dimostrazioni, solo una piccola nota @5:07 affermi "con resto 3", in realtà è un refuso xchè si vede che è "con resto 2".
    Mi piace molto l'eleganza della seconda dimostrazione con le congruenze

    • @GaetanoDiCaprio
      @GaetanoDiCaprio  Před rokem

      Hai ragione, ho notato subito quel refuso dopo poco che avevo caricato il video. Grazie!!

  • @tcvadomix
    @tcvadomix Před rokem

    La divisibilità è ben definita per i numeri irrazionali? Altrimenti basterebbe dire che n^2+1=(n-i)(n+i) e il coefficiente di i non è divisibile per 3 in entrambi.

    • @GaetanoDiCaprio
      @GaetanoDiCaprio  Před rokem

      Forse intendevi i numeri complessi? In ogni caso la divisibilità è ovviamente argomento che riguarda esclusivamente i numeri interi, dai razionali in poi la divisione è sempre possibile (eccetto quando il divisore è zero).

    • @tcvadomix
      @tcvadomix Před rokem

      @@GaetanoDiCaprio Sì, mi scusi, distrazione.

  • @tere7356
    @tere7356 Před rokem

    Forse si potrebbe risolvere anche con un equazione di 2 grado dove pongo
    n²+1=3n
    n²-3n+1=0
    ×=3+- radice q 9+4/2
    ×=3+-radice q 13/2
    Il risultato nn potrà mai essere un N

    • @GaetanoDiCaprio
      @GaetanoDiCaprio  Před rokem

      Errato. L'equazione sarebbe n^2+1=3k, ossia un'equazione in DUE variabili.

  • @Chiavaccio
    @Chiavaccio Před rokem

    Grande!

  • @giulial
    @giulial Před rokem

    Buongiorno sono d'accordo con math_art commento di 4 giorni fa.
    Infatti rad2 non è intero. Va bene, come dice lei: la somma di razionali più essere un intero, ma in questo caso il razionale è 8/3quindi n = 2radq 2 , cioè n^2=8 non esiste per n intero. Il fatto che 8/3+2/3= intero non è sufficiente per escludere la dimostrazione, perché 8 non è quadrato (ma cubo al limite di n=2)

    • @giulial
      @giulial Před rokem

      8/3+1/3 correggo...

    • @GaetanoDiCaprio
      @GaetanoDiCaprio  Před rokem

      Mi dispiace la dimostrazione non è corretta. Il valore 8/3 è puramente arbitrario, non c'entra nulla con la dimostrazione. Le frazioni e le radici quadrate servono poco quando si dimostra sugli interi. Occorre ragionare con somma/sottrazione o prodotto, oppure con le congruenze (per chi ha voglia di imparare qualcosa)

    • @giulial
      @giulial Před rokem

      @@GaetanoDiCaprio naturale che 8/3 fosse un esempio...ma il fatto che si ottenga un multiplo di radq di 2 implica la negazione delle ipotesi di n naturale

    • @GaetanoDiCaprio
      @GaetanoDiCaprio  Před rokem

      @@giulial Allora, visto che insisti (fai bene, bisogna farsi valere!) scrivi per benino TUTTA la tua dimostrazione, così riesco a risponderti in maniera più convincente (spero)

    • @giulial
      @giulial Před rokem

      @@GaetanoDiCaprio scusi magari sbaglio...purtroppo non posso mostrare il mio foglio e da tastiera faccio fatica:
      Divido (n^2+1) per 3, spezzo, pongo uguale a k intero.
      Porto a sinistra 1/3 e cambio segno.
      Per k =1 n^2 =2 quindi irrazionale quindi impossibile
      Per k = 2 allora n^2 =5 quindi impossibile n naturale
      Per k =3 allora n^2 = 8 quindi ancora n non naturale
      Dunque senza procedere all'infinito
      Guardiamo a sinistra dell'uguale:
      Abbiamo k - 1/3 dove k è naturale
      Facciamo denominatore comune 3
      Quindi moltiplichero' k per 3 , quindi avrò un multiplo di 3, ma poi dovrò sottrarre 1 a questo multiplo, perché devo sottrarre l'1/3 . Quindi il numeratore non sarà più multiplo di 3. Esso è uguale a n^2 , che dunque non sarà multiplo di 3. Ora devo dimostrare che questo numero non multiplo di 3 pari a 3k-1 non è un quadrato, così vedrò che n non è naturale ma irrazionale. Quindi 3k-1 lo pongo uguale a un quadrato generico pari a q per q = q^2.
      Divido per q e trovo (3k-1)/q = q
      Quindi se q fosse uguale a 1
      3k-1=1 cioè k = 2/3 ma avevo posto k naturale o al massimo intero.
      Sono arrivata solo qui per ora mi dispiace, ma dopo per q diversi da 1 trovo 3k = 1 +q^2, quindi 3k non è un quadrato e un 'non quadrato' meno 1 non è detto che non sia un quadrato. Ad esempio 5-1 =4 che è quadrato di 2, però 5 non è un 3k perché 5 non è divisibile per 3 e k è intero, quindi non è un esempio correlato. Ci penserò ancora su...grazie e scusi il disturbo. Non si preoccupi di rispondere, stavo solo cercando di completare la mia strada che vedevo simile ad altri tentativi, ma evidentemente è un vicolo cieco. Grazie!

  • @peterdecupis8296
    @peterdecupis8296 Před rokem

    tutto sommato, la seconda soluzione è semplicemente la prima riformulata nell'algebra modulare, le cui proprietà sono integralmente fondate sul teorema Quoziente/Resto valido in N e Z

  • @francescopalladino8982
    @francescopalladino8982 Před rokem +1

    Prima di guardare il video provo. Abbiamo 3 casi possibili: il primo è che n sia multiplo di 3, in questo caso anche n^2 è multiplo di 3 e quindi n^2 + 1 non lo è. Il secondo caso è che n sia del tipo 3n+1. In questo caso n^2 è 9n^2 + 6n + 1 e quindi n^2 + 1 è 9n^2 + 6n + 2 che non è multiplo di 3. Ultimo caso è che n sia del tipo 3n+2. Qui n^2 è 9n^2 + 12n + 4 e quindi n^2 + 1 è 9n^2 + 12n + 5 che non è multiplo di 3. La risposta è che non è mai divisibile per 3

  • @mcumer
    @mcumer Před 3 měsíci

    Con le classi resto modulo 3 si vede subito che è vero.. ma i candidati le studieranno al primo anno

  • @giacomomiceli3082
    @giacomomiceli3082 Před rokem

    più che un'alternativa dimostrazione al seguente quesito, ne avrei un'altro, ovvero dato n naturale è vero che [(11^n) - 1] è sempre divisibile per 10?

    • @GaetanoDiCaprio
      @GaetanoDiCaprio  Před rokem +3

      Sì è vero e si può dimostrare in tanti modi. Il modo più semplice e elementare è osservare che una potenza di 11 ha sicuramente 1 come cifra delle unità

    • @andream61
      @andream61 Před rokem

      x^n-y^n è sempre divisibile per x-y. Dunque 11^n-1=11^n-1^n è divisibile per 11-1=10.

    • @GaetanoDiCaprio
      @GaetanoDiCaprio  Před rokem +1

      @@andream61 ottimo

  • @robertopicco1972
    @robertopicco1972 Před rokem

    Si può scrivere n = (10a+b) => n²+1 = 100a² + 10(2ab) + b² +1. Avrò quindi a² centinaia, (2ab) decine e (b²+1) unità. Affinché un numero sia divisibile per tre, la somma delle sue cifre deve essere divisibile per 3, quindi essere 3, 6 o 9, quindi (a²+2ab+b²) +1 = 3,6,9 => (a+b)² = 2,5,8 ma nessun quadrato somma di numeri interi fa 2,5,8 quindi n²+1 non è divisibile per 3.

    • @GaetanoDiCaprio
      @GaetanoDiCaprio  Před rokem

      Molto interessante ma così dimostri l'asserto soltanto per n compreso tra 10 e 99, non per TUTTI gli n interi. Grazie comunque per l'idea che potrebbe essere utile per altre dimostrazioni

    • @robertopicco1972
      @robertopicco1972 Před rokem

      Secondo me non è così. Prova a mettere degli interi a e b qualunque, anche negativi, e vedrai che funziona :-)

    • @GaetanoDiCaprio
      @GaetanoDiCaprio  Před rokem +1

      @@robertopicco1972 Nella tua dimostrazione a e b rappresentano le cifre di un un numero di due cifre. Se non è così la dimostrazione è errata

  • @simgallgallia6848
    @simgallgallia6848 Před rokem +1

    Con il principio di induzione non si riuscirebbe?

    • @GaetanoDiCaprio
      @GaetanoDiCaprio  Před rokem

      Io non ci sono riuscito

    • @maurorusso4253
      @maurorusso4253 Před rokem

      @@GaetanoDiCaprio poco fa ho inserito a commento un ragionamento che mi sembra induttivo, sebbene ho usato il termine 'ricorsione' 😅 per abitudine informatica

    • @vittorioemanueleferrante2734
      @vittorioemanueleferrante2734 Před rokem

      @@GaetanoDiCaprio Il passo induttivo è (n+1)^2+1 = n^2 + 1 + 2n +1. L'ipotesi induttiva lascia un resto di 1 o di 2 su 2n+1. Nel primo caso è un numero pari; nel secondo è un numero pari + 3, che non è divisibile.

  • @marcoclocchiatti4772
    @marcoclocchiatti4772 Před rokem

    si può fare anche per induzione.
    per n=0, N=n^2+1 ha resto 1.
    se n->n+1, N->N+2n+1.
    se n è divisibile per 3, 2n+1 ha resto 1. Quindi N(1) ha resto 2.
    se n è divisibile per 3, con resto 1, 2n+1 ha resto zero. Quindi N(2) ha nuovamente resto 2.
    se n è divisibile per 3 con resto 2, 2n+1 ha resto 2. Questa volta N(3) ha resto 2+2 ... cioè 1.
    alla terza volta il ciclo si ripete.

    • @GaetanoDiCaprio
      @GaetanoDiCaprio  Před rokem

      Ciao, mi pare corretta, ma (non so se sei d'accordo) non è particolarmente più elegante o più concisa delle altre dimostrazioni, è sostanzialmente dello stesso stile per casi. Grazie mille!

    • @marcoclocchiatti4772
      @marcoclocchiatti4772 Před rokem

      @@GaetanoDiCaprio sono d'accordo. non è affatto più elegante delle altre, perché le dimostrazioni eleganti le avete già fatte voi. :) però mi sembra utile a osservare che si tratta di una proprietà induttiva. non è dello stesso stile delle dimostrazioni per casi, perché il funzionamento dipende dall'ordine delle operazioni. se lo zero fosse seguito da un numero con resto 2, passerei direttamente da 1 a 6 e il teorema cadrebbe.

    • @GaetanoDiCaprio
      @GaetanoDiCaprio  Před rokem

      @@marcoclocchiatti4772 Non volevo sminuire la tua dimostrazione! E' ottima, era solo una considerazione "estetica". Grazie di nuovo!!

    • @marcoclocchiatti4772
      @marcoclocchiatti4772 Před rokem

      @@GaetanoDiCaprio grazie anche a te. non ti ho risposto per paura di essere sminuito, ma perché la tua osservazione mi ha stimolato ad approfondire il concetto da un altro punto di vista e a comprenderlo meglio. a volte l'estetica lavora al contrario. l'induzione è uno strumento primitivo. quindi una dimostrazione per induzione è un po' come un programma in assembler. così le dimostrazioni per induzione sono esteticamente meno apprezzabili, ma nello stesso tempo colgono maggiori sfumature di significato.

  • @pivxtrex8698
    @pivxtrex8698 Před rokem

    Non ho capito il passaggio in cui dici che "questo numero diviso per tra da come resto 1" al minuto 2:36 - quale numero?

    • @GaetanoDiCaprio
      @GaetanoDiCaprio  Před rokem

      Il numero (3k)^2+1

    • @pivxtrex8698
      @pivxtrex8698 Před rokem

      @@GaetanoDiCaprio Grazie - in pratica deve risultare che la prima parte di espressione mi da resto zero (quindi divisibile) in modo da ottenere sempre resto 1?

    • @GaetanoDiCaprio
      @GaetanoDiCaprio  Před rokem

      @@pivxtrex8698 se un numero ha la forma 3(...)+r e 0

  • @_zack_4857
    @_zack_4857 Před rokem

    Puoi fare anche i quesiti degli anni più recenti? Ad esempio quello di quest'anno il 3

    • @GaetanoDiCaprio
      @GaetanoDiCaprio  Před rokem +1

      Magari fossi in grado di risolvere tutti i quesiti della Normale!

    • @_zack_4857
      @_zack_4857 Před rokem

      @@GaetanoDiCaprio 😅

    • @danielesommo
      @danielesommo Před 11 měsíci

      @@GaetanoDiCaprio secondo me potrebbe essere interessante anche solo un video in cui i mostri e le criticità che non ti permettono di risolverlo, si potrebbero creare una sorta di gang risolvi problemi fra i tuoi iscritti

  • @stefanodellicarpini7293

    Una domanda: dato che la domanda è sulla possibilità di essere divisibile per 3 non si potrebbe fare un ragionamento del tipo "n² +1 = 3" e dopo averlo risolto notare che n = 1,414 non fa mai un numero intero se moltiplicato per un numero intero? O ho sbagliato qualcosa?

    • @GaetanoDiCaprio
      @GaetanoDiCaprio  Před rokem +5

      Sì, ci sono almeno i seguenti errori:
      -- divisibile per 3 non vuol dire UGUALE a 3;
      -- 1,414 x 1000 = 1414 che è un numero intero.
      -- la radice quadrata di 2 non è uguale a 1,414: è un numero irrazionale, la sua espansione decimale è infinita senza periodo.
      La tua idea, tuttavia, non è totalmente errata: potrebbe essere la base di una dimostrazione per assurdo. Se n^2+1 fosse multiplo di 3 allora esisterebbe un intero k tale che n^2+1=3k, ossia n^2 = 3k-1. Ma allora 3k-1 dovrebbe essere un quadrato perfetto, ma questo non può accadere (da dimostrare!), quindi n^2+1 non può essere divisibile per 3

    • @stefanodellicarpini7293
      @stefanodellicarpini7293 Před rokem +1

      @@GaetanoDiCaprio Ah, grazie mille

    • @voltydequa845
      @voltydequa845 Před rokem

      @@GaetanoDiCaprio Lo spunto del "per assurdo" m'ha fatto venire in mente una variante:
      k = n^2/3 + 1/3, ovvero n^2 deve essere di mod 2, ma non può esserlo dato che se n è divisibile per tre abbiamo mod 0, e se non è divisibile abbiamo mod 1 in virtù del n^2 - 1 sempre divisibile per 3 (ove n non divisibile per tre).
      p.s. dopodiché chiudo - ché la matematica è bella finché non dura troppo :)

  • @umbertocappellazzo19
    @umbertocappellazzo19 Před rokem

    Ciao, secondo te sarebbe possibile dimostrare il quesito per induzione? Ipotizziamo che l'enunciato sia valido per n, e vogliamo dimostrare che è valido anche per n+1. Il passo zero è banalmente verificato per sostituzione: N = 0^2 +1 = 1 che non è divisibile per 3. Se consideriamo il passo induttivo, e quindi per n l'enunciato sia vero, e vediamo cosa succede per n+1: N = (n+1)^2 +1 = n^2 +1 + 2n + 1. n^2 +1 è vero per ipotesi, però poi bisognerebbe dimostrare che la combinazione di n^2 +1 e 2n +1 non è divisibile per 3. Qualche idea a riguardo?

    • @GaetanoDiCaprio
      @GaetanoDiCaprio  Před rokem

      Io non ho trovato una dimostrazione per induzione. Tu?

    • @andreaforte976
      @andreaforte976 Před rokem +1

      Sì, è possibile dimostrare il quesito per induzione, ma credo che con il tuo metodo tu ti stia un po' complicando la vita.
      Probabilmente è possibile farlo anche con la combinazione di n^2 +1 e 2n +1, ma usando n+3 anziché n+1 puoi sfruttare a tuo favore il fatto che il resto della divisione di N^2+1 per 3 ha un andamento periodico (1,2,2, 1,2,2, 1,2,2...).
      Ecco la dimostrazione per induzione a cui ho pensato:
      Tramite sostituzione dimostro facilmente che N^2+1 non è multiplo di 3 per N = 0, 1 e 2.
      Dopodiché dimostro che se la mia affermazione è valida per un generico N allora deve valere anche per N+3
      Ipo: (N)^2 +1 non è multiplo di 3
      Tesi: (N+3)^2 +1 non è multiplo di 3
      Dim:
      (N+3)^2 +1 = ( N^2 +6N +9 ) +1 = (N)^2 +1 +(6N+9) = N^2 +1 +3(2N+3)
      Poiché 3(2N+3) è un multiplo di 3, allora sommandolo ad un generico numero, quest'ultimo non modifica il resto della sua divisione con 3.
      Quindi se N^2 +1 non è multiplo di 3 neanche (N+3)^2 +1 ( che è pari a N^2 +1 +3(2N+3) ) lo sarà.
      QED

    • @umbertocappellazzo19
      @umbertocappellazzo19 Před rokem

      @@andreaforte976 Non sono sicuro al 100%, ma in teoria se vuoi dimostrare per induzione devi considerare il passo n e il passo n+1, quindi non puoi usare come passo induttivo n+3. Facendo così ti perdi i numeri n+1 e n+2, quindi tu stai dimostrando il fatto solo per un sottoinsieme di numeri. Non so cosa ne pensi Gaetano a riguardo.
      Comunque ero curioso di sapere se tramite la dimostrazione per induzione fosse plausibile, può essere che diventi troppo complesso, ma solitamente questo genere di quesiti si sposa bene con il principio di induzione

    • @GaetanoDiCaprio
      @GaetanoDiCaprio  Před rokem

      @@andreaforte976 Bella idea! Peccato che il ragionamento finale sia scorretto: nulla mi dice che aggiungendo un "generico numero" questo non modifica il resto della divisione per 3. Ad esempio 5 diviso 3 dà resto 2, ma se aggiungo 1 dà resto zero!

    • @andreaforte976
      @andreaforte976 Před rokem

      @@GaetanoDiCaprio Penso che la mia frase sia stata un po' fraintesa, probabilmente sono stato poco chiaro.
      Io non sto aggiungendo un generico numero. Sto aggiungendo un multiplo di 3 AD un generico numero (che nel mio caso corrisponde ad N^2+1 ) ed è quest'ultimo a non modificare il suo resto della divisione per 3 anche dopo la somma.
      Ad esempio se 5 mod 3 =2 allora per ogni kEZ, anche (5 +3k) mod 3=2, ovvero 8 mod 3 = 2, 11 mod 3 = 2, 14 mod 3 = 2, 17 mod 3 = 2, ect...
      (Sto sommando un multiplo di 3 ad il generico numero)
      Nella mia dimostrazione il generico numero era N^2+1 al quale sommo un multiplo di 3 (ovvero +3(2N+3) )
      qualunque sia il resto della divisione di N^2+1 per 3 sommandoci un multiplo di 3 il resto non cambia
      ( N^2+1 +0 )mod3 = ( N^2+1 +3 )mod3 = ( N^2+1 +6 )mod3 = ( N^2+1 +9 )mod3 = ( N^2+1 +3(2N+3) )mod3

  • @lucagiovannini5096
    @lucagiovannini5096 Před rokem

    e rislovere l' equazione ((n^2)+1)/3 = 0 ? viene che n^2= -1 che è impossibile...

    • @GaetanoDiCaprio
      @GaetanoDiCaprio  Před rokem +1

      Divisibile per 3 non vuol dire che se dividiamo per 3 otteniamo come quoziente zero. Vuol dire che se dividiamo per 3 otteniamo RESTO zero

  • @vladyslavchornous5035

    Scusa io ho fatto il seguente ragionamento: (n^2+1)/3 = n^2/3+1/3 quindi qualunque intero positivo sostituisco ad n questo sarà maggiorato sempre di 1/3 e quindi mai un intero e quindi mai divisibile

    • @gianfrancoimperiale9698
      @gianfrancoimperiale9698 Před rokem

      Tu parti dal presupposto che ogni n^2 sia divisibile per 3. Il che non è manifestamente vero

    • @GaetanoDiCaprio
      @GaetanoDiCaprio  Před rokem +2

      Se prendi 8/3 e la aumenti di 1/3 ottieni come risultato 3. Il fatto che una frazione sia "maggiorata" di 1/3 non implica che il risultato non sia intero

    • @vladyslavchornous5035
      @vladyslavchornous5035 Před rokem

      Perfetto!

  • @maurorusso4253
    @maurorusso4253 Před rokem

    a) se n è dispari, allora n^2 + 1 è pari e non divisibile per 3
    b) se n è pari, allora vale 2k, per cui n^2 + 1 = 4*k^2 + 1, che in modulo 3 è congruente a k^2 + 1, per cui condizione necessaria è che sia divisibile per 3 anche k^2 + 1. Ma questo provoca una ricorsione fra numeri dimezzati fino inevitabilmente a sbattere su un numero dispari che rientra nel caso a) e quindi a ritroso anche il caso b) è certamente fallace.
    Ammenda postuma: il caso a) è erroneo, pardon.

    • @GaetanoDiCaprio
      @GaetanoDiCaprio  Před rokem +1

      Non è un ragionamento induttivo, distinguere tra pari e dispari non è induttivo. Ottimo tentativo comunque

    • @maurorusso4253
      @maurorusso4253 Před rokem

      @@GaetanoDiCaprio mi riferivo al caso b) dove si basa sul risultato di un numero più piccolo, Certo, non un puro n -> n+1, con tutti i dispari come caso base.... ops, qui ho sbagliato, non è vero che se n è dispari allora n^2 + 1 non è divisibile per 3 😅🤦‍♂️. Va beh, però riuscito a dire che il problema si riduce al solo caso di n dispari.

    • @annacerbara4257
      @annacerbara4257 Před rokem +1

      caso a): se un numero è pari può essere divisibile per 3, esempio: 6

    • @GaetanoDiCaprio
      @GaetanoDiCaprio  Před rokem

      @@annacerbara4257 esatto

    • @maurorusso4253
      @maurorusso4253 Před rokem +1

      @@annacerbara4257 sì sì, avevo infatti successivamente fatto ammenda 😅

  • @boclock382
    @boclock382 Před rokem

    Io avrei pensato di esplicitare direttamente la soluzione, cioè n^2/3 +1/3. Per essere intera questa quantità deve avere n^2 pari a m+2/3 cosa evidentemente impossibile se n è intero

    • @GaetanoDiCaprio
      @GaetanoDiCaprio  Před rokem

      Cos'è m?

    • @boclock382
      @boclock382 Před rokem

      @@GaetanoDiCaprio un altro intero

    • @boclock382
      @boclock382 Před rokem

      @@GaetanoDiCaprio poi diciamo che la mia dimostrazione forse ha molto poco di formalismo matematico ahaha

    • @GaetanoDiCaprio
      @GaetanoDiCaprio  Před rokem

      @@boclock382 non è questione di formalismo è che è un ragionamento non corretto. Perché m deve essere intero?

    • @alessandrotommasi9941
      @alessandrotommasi9941 Před rokem

      Certo che è corretta. Se n^2+1 è divisibile per 3, allora n^2/3+1/3 è intero, il che significa che n^2/3 deve poter essere scritto nella forma m+2/3 con m intero. Siccome deve essere n^2 = m+2/3, ma n^2 è intero per n intero, non esiste soluzione. È la stessa dimostrazione che avevo pensato io.

  • @stefanodudine8653
    @stefanodudine8653 Před rokem

    Per induzione:
    Se (n-1)^2+1=n^2+1-2n+1 non è multiplo di 3 e n^2+1 non è multiplo di 3
    e
    (n+1)^2+1=n^2+1+2n+1=(n-1)^2+1+4n è multiplo di 3
    => 2n+1 e 4n non sono multipli di 3.
    Se lo fossero allora (n+1)^2+1 sarebbe la somma tra un non multiplo e un multiplo di 3, che non può essere un multiplo di 3.
    Se 2n+1=3q+1
    => 4n=6q
    => 4n è multiplo di 3.
    Se 2n+1=3q+2
    => (n+1)^2+1=n^2+2n+2=n^2+3(q+1)
    => n^2 è multiplo di 3
    => n è multiplo di 3
    => 4n è multiplo di 3.

  • @allittofrancesco6072
    @allittofrancesco6072 Před rokem

    Ho svolto una dimostrazione come la prima, solo che il numero n l'ho considerato come 3k, 3k+1 e 3k-1 e non 3k+2. Dopo il ragionamento è identico

  • @irtizio
    @irtizio Před rokem

    perchè devo sostituire n con 3k?

    • @GaetanoDiCaprio
      @GaetanoDiCaprio  Před rokem

      I multipli di 3 (i numeri che divisi per 3 danno resto 0) hanno la forma 3k, dove k è un intero.

    • @irtizio
      @irtizio Před rokem

      ok, comunque io lho risolta vedendo due cose:
      la prima è sapendo che x non può essere un numero divisibile per 3, visto che al quadrato sarà sempre divisibile per 3 ma, solo per il fatto che laaddizioni con 1, la somma non sarà divisibile per tre (esempio (3²)+1= 10 è cosí via).
      poi se invece metto a x tutti quei numeri non divisibile per 3 noto che nonostante io aggiungo 1 la somma non diverrà divisibile per 3 esempio 1²+1= 2, 2²+1=5, 4²+1=17, 5²+1= 26 (si noti come con x=1 il +1 non basti per farlo diventare 3 mentre in tutti gli altri x=n(diverso da divisori di 3) il + 1 non sia abbastanza per far sí che il numero sia divisibile per 3.

    • @GaetanoDiCaprio
      @GaetanoDiCaprio  Před rokem

      @@irtizio sostituire qualche numero serve solo a farsi un'idea, non è una dimostrazione.

  • @filippomontonati3428
    @filippomontonati3428 Před rokem

    Io ho considerato n^2 + 1 congruo a -2*n^2 + 1 mod (3).
    (infatti n^2 + 1 - (-2*n^2 +1) = 3*n^2 che è multiplo di 3 per ogni n naturale).
    Quindi per avere n^2+1 congruo a 0 mod(3) bisogna solo impostare l'equazione -2*n^2 + 1 = 0 che per n naturale non ha soluzione.

    • @GaetanoDiCaprio
      @GaetanoDiCaprio  Před rokem

      Interessante ma la conclusione della dimostrazione è errata. In sostanza hai "spostato" il problema di dimostrare che n^2+1 non è mai congruo a zero modulo 3 nel problema di dimostrare che -2n^2+1 non è mai congruo a zero modulo 3... Non c'è un gran vantaggio

    • @filippomontonati3428
      @filippomontonati3428 Před rokem

      @@GaetanoDiCaprio Avevo l'impressione che qualcosa nella dimostrazione non andasse (premetto che non sono esperto dell'argomento). Tuttavia, potrebbe essere più specifico riguardo all'errore che avrei commesso? La ringrazio :)

    • @GaetanoDiCaprio
      @GaetanoDiCaprio  Před rokem

      @@filippomontonati3428 Sono stato molto specifico: non basta "impostare l'equazione" con l'uguaglianza a zero, ma con la congruenza a zero

  • @manuelpassarella
    @manuelpassarella Před rokem

    Se noi ponessimo n^2 + 1 = 3x
    Allora (n^2 + 1)/3 = x
    Allora n^2/3 + 1/3 = x
    X quindi ha sempre una parte decimale. Quindi n^2 + 1 non può mai dare un numero intero perfettamente divisibile per tre.

    • @GaetanoDiCaprio
      @GaetanoDiCaprio  Před rokem

      No, il ragionamento è errato. La somma di due frazioni può benissimo essere un intero, ad esempio 8/3+1/3=3

    • @manuelpassarella
      @manuelpassarella Před rokem

      @@GaetanoDiCaprio Giusto! Sono un po' arrugginito

  • @idemconpatate7111
    @idemconpatate7111 Před rokem +1

    carino ma ha dimenticato la soluzione più semplice e nel primo minuto del video era già oltre al 60% della dimostrazione.
    un qualsiasi numero è divisibile x 3 quando la somma dei suoi numeri da 3 o un suo multiplo (6 e 9), bastava continuare con: (7^2+1) , (8^2+1) , (9^2+1)
    nessuno divisibile x 3. soluzione al quesito ottenuta in 5 secondi

    • @GaetanoDiCaprio
      @GaetanoDiCaprio  Před rokem +2

      Grazie per il suggerimento ma non sono d'accordo. Quegli esempi all'inizio non hanno nulla a che vedere con le cifre: i tre esempi aggiuntivi che secondo lei completerebbero la dimostrazione non hanno nulla a che vedere col criterio di divisibilità per 3 da lei citato.

    • @idemconpatate7111
      @idemconpatate7111 Před rokem

      grazie per la risposta, ma non sono d' accordo , se prendiamo qualsiasi numero e ci mettessimo a sommare tutte le cifre alla fine arriveremo semplicemente ad un numero compreso tra 1 e 9.
      non ho messo in dubbio la correttezza delle dimostrazioni, ma durante i test si deve trovare la soluzione il prima possibile e nel minore tempo, il test non richedeva la dimostrazione della risposta ma semplicemente chiede se tutti i (numeri^2+1)/3 danno come risultato un numero intero oppure no. Volendo essere ancora più pignoli si potevano escludere 3 , 6 e 9 immediatamente; 1 e 2 facendo un rapidissimo calcolo mentale; rimanendo semplicemente col 4 , 5 , 7 e 8 ; arrivando alla soluzione del quesito in 10/15 secondi.
      chiudo rispondendo alle ultime righe nella sua risposta: un qualsiasi numero è divisibile x 3 quando la somma di tutti i numeri di cui è compreso da come risultato 3 o un suo multiplo 6 o 9.

    • @GaetanoDiCaprio
      @GaetanoDiCaprio  Před rokem +5

      @@idemconpatate7111 Il criterio di divisibilità per 3 lo conosco, grazie, ma la sua presunta "dimostrazione" non lo usa affatto, o lo usa in un modo che non esplicita alcune proprietà. Se vuole tentare una dimostrazione che usa il criterio di divisibilità si deve chiedere: che relazione c'è tra la somma delle cifre di un numero e la somma della cifre DEL NUMERO AL QUADRATO AUMENTATO DI 1. Può essere una dimostrazione alternativa ma non è affatto ovvia, occorre comunque giustificarla con le congruenze.

    • @mrkilowatt1811
      @mrkilowatt1811 Před rokem +1

      Ma scusami ma dove sta scritto che n deve stare tra 0 e 9, potrebbe tranquillamente essere un numero più alto

    • @mrkilowatt1811
      @mrkilowatt1811 Před rokem

      @@idemconpatate7111 penso che tu non abbia capito a fondo il quesito

  • @ninoporcino5790
    @ninoporcino5790 Před rokem

    con le congruenze è molto più facile, perché non si insegnano alle scuole superiori mi chiedo

    • @GaetanoDiCaprio
      @GaetanoDiCaprio  Před rokem +2

      Forse perché la teoria sottostante non è semplicissima. Ma la motivazione più plausibile è che la maggior parte dei docenti non conosce le congruenze: che io sappia sono obbligatorie soltanto nel corsi laurea di matematica e informatica. Moltissimi docenti di matematica sono laureati in fisica o in ingegneria

  • @michellaboureur7651
    @michellaboureur7651 Před rokem

    Se n^2 + 1 = (modulo 3), allora pure n^2 - 2= (modulo 3) cioè (n + rad2) (n - rad2) = (modulo 3) che è assurdo visto che non essendo interi nessuno di quei due fattori è divisibile per 3 con quoziente intero

    • @GaetanoDiCaprio
      @GaetanoDiCaprio  Před rokem

      Incomprensibile

    • @michellaboureur7651
      @michellaboureur7651 Před rokem

      @@GaetanoDiCaprio dimostrazione per assurdo.Se n^2 + 1 è divisibile per 3 allora n^2 + 1 = 0 (mod 3) che implica necessariamente e reciprocamente che n^2 - 2 = (n + rad2) (n - rad2) = = 0 (mod 3). Siccome né (n + rad2) = 0 (mod 3) né (n - rad2) = 0(mod 3) non essendo interi, neanche il loro prodotto è = 0(mod3) cioè divisibile per 3.(non proponendo la mia tastiera i segni adatti, = significa congruenza e rad2 radice quadrata di 2)

    • @GaetanoDiCaprio
      @GaetanoDiCaprio  Před rokem

      @@michellaboureur7651 Adesso ho capito grazie. Mi dispiace, la dimostrazione è errata, la scomposizione con i radicali non ci dà alcuna informazione sui divisori interi del numero n^2-2.

    • @michellaboureur7651
      @michellaboureur7651 Před rokem

      @@GaetanoDiCaprio grazie per l attenzione e per questi questi che stimlolano la riflessione anche se non sempre a buon fine matematico come illustra il mio errore. Auguri

    • @GaetanoDiCaprio
      @GaetanoDiCaprio  Před rokem

      @@michellaboureur7651 L'errore è il presupposto inevitabile dell'apprendimento. Auguri a te e grazie per l'attenzione ai miei video

  • @massimos8050
    @massimos8050 Před rokem

    bisognerebbe formulare meglio i quesiti. n^2+1 è sempre divisibile per 3 dipende dal resto.

  • @anto8375
    @anto8375 Před rokem

    Io ho usato il principio di induzione, sono pigro.
    Ho controllato se la formula vale nel caso base (no), ho supposto che esista una n-1 per cui sia vera, ho controllato se la formula vale per n (no).
    Quindi andando a ritroso trovo che il caso n-1 diventa/arriva 1 (per cui non vale).

  • @sergiobuschi4201
    @sergiobuschi4201 Před rokem

    Bata considerare la mappa norfismo quoziente di anelli: Z -> Z/(3).

    • @GaetanoDiCaprio
      @GaetanoDiCaprio  Před rokem

      Grazie del contributo, se riuscissi a dettagliarlo meglio sarebbe utile

  • @giovanniautobello2677
    @giovanniautobello2677 Před rokem +1

    9K^2+12k+3+2=3*(3K^2+4k+1)+2

  • @MarcoBigiarini-fo1ix
    @MarcoBigiarini-fo1ix Před rokem

    Anche per induzione .....

    • @GaetanoDiCaprio
      @GaetanoDiCaprio  Před rokem

      Come? Puoi dettagliare? Grazie

    • @vittorioemanueleferrante7107
      @vittorioemanueleferrante7107 Před rokem

      @@GaetanoDiCaprio L'ho già scritto qui altrove.
      Per induzione, il passo induttivo sta nel dimostrare che è vero per n+1, quando è vero per n. Quindi che è vero per (n+1)^2 + 1. Ma questo è uguale a n^2 + 1 + 2n +1. Dato che è vero per n, allora la divisione di n^2+1 lascia un resto di 1 o di 2, che va sommato a 2n+1. Nel primo caso ne sorte un numero pari, che non è divisibile per 3; nel secondo, ne sorte un numero pari più 3, che ovviamente non è divibile per 3.

  • @matteomunafo4372
    @matteomunafo4372 Před rokem +1

    Per il teorema di Natale dato p primo esiste g tc g^2=-1 mod p se e solo se p=1 mod 4 spoiler 3 = 3 mod 4 quindi non esite n

  • @francescopaolorussi9016

    Radice di 2 al quadrato + 1 fa tre divisibile per 3

    • @GaetanoDiCaprio
      @GaetanoDiCaprio  Před rokem

      n deve essere un numero intero, conviene guardare un video prima di lasciare commenti

  • @giuliofalco9816
    @giuliofalco9816 Před rokem

    La dimostrazione che ho trovato è la seguente. Suppongo per assurdo che il numero n^2 + 1 sia divisibile per 3
    ovvero che la congruenza n^1 + 1 = 0 mod 3 sia soddisfatta per qualche n
    il numero n^2 che è intero, nell'aritmetica modulo 3 può essere congruo solo a 0,1,2. Solo il valore 2 è possibile gli altri rendono la congruenza falsa, contrariamente all'ipotesi, Quindi n^2 = 2 + 3k ossia n n - 2 = 3 k e k deve essere intero. Anche se 3 fosse divisore di n, non lo è di sicuro di 2. Quindi non esiste nessuna k intero che soddisfa questa condizione. L'ipotesi è dunque falsa

    • @GaetanoDiCaprio
      @GaetanoDiCaprio  Před rokem +1

      Quando dici "Anche se 3 fosse divisore di n, non lo è di sicuro di 2" è corretto ma a mio avviso da quell'affermazione non puoi dedurre nulla riguardo alla divisibilità per 3 dell'espressione n^2-2

    • @giuliofalco9816
      @giuliofalco9816 Před rokem

      @@GaetanoDiCaprio
      Ho immaginato di dividere ambo i membri dell'ultima espressione per 3. A destra rimane k che deve essere intero per ipotesi. Di conseguenza ciò che sta a sinistra deve essere anche lui intero visto che c'è un'uguaglianza di mezzo. Ma la frazione la posso scomporre nell differenza di due frazioni. La prima può essere un intero ma la seconda frazione no 2/3. Di conseguenza il risultato non può essere intero

    • @GaetanoDiCaprio
      @GaetanoDiCaprio  Před rokem +3

      ​@@giuliofalco9816 quello che volevo dire è che se un numero non è divisore di nessuno di due addendi, potrebbe tranquillamente essere divisore della somma: esempio 3 non è divisore di 4 e non è divisore di 2 ma è divisore di 4+2

    • @giuliofalco9816
      @giuliofalco9816 Před rokem +1

      @@GaetanoDiCaprio Già, la conclusione che ho tratto è errata effettivamente. Non si può fare un ragionamento così con la somma, hai ragione. Solo con il prodotto. Quindi l'unica è analizzare i possibili valori di n, come nel video e concludere che visto che 0^1 = 0, 1^2 = 1 e 2^2 = 1 mod 3 la congruenza n^2 + 1 = 0 mod 3 non è mai soddisfatta

    • @michellaboureur7651
      @michellaboureur7651 Před rokem +1

      Non avendo pazienza di vedere tutti I commenti precedenti, questo mio forse sarà ridondante : se n^2 + 1 = (mod 3) allora n^2 - 2 = (mod 3) cioè (n + rad 2) (n - rad 2) = (mod 3) ; siccome né (n + rad 2) né (n - rad 2) sono (mod 3) in quanto non interi si dimostra per assurdo che n^2 + 1 non è mai (mod 3). Meno elegante ma probante ugualmente.

  • @vsalvato53
    @vsalvato53 Před rokem

    It is possible for "n to the 2nd exp + 1" to be divisible by 3. However, it is not always divisible by 3.
    To determine whether a number is divisible by 3, you can check whether the sum of its digits is divisible by 3. For example, consider the number "n to the 2nd exp + 1" when n is equal to 2:
    2 to the 2nd exp + 1 = 4 + 1 = 5
    The sum of the digits of 5 is 5, which is not divisible by 3. Therefore, 5 is not divisible by 3.
    On the other hand, consider the number "n to the 2nd exp + 1" when n is equal to 3:
    3 to the 2nd exp + 1 = 9 + 1 = 10
    The sum of the digits of 10 is 1 + 0 = 1, which is not divisible by 3. Therefore, 10 is not divisible by 3.
    In general, "n to the 2nd exp + 1" will not be divisible by 3 unless the value of "n" is itself divisible by 3. For example:
    6 to the 2nd exp + 1 = 37, which is not divisible by 3
    9 to the 2nd exp + 1 = 82, which is divisible by 3
    12 to the 2nd exp + 1 = 145, which is not divisible by 3

  • @4324swer
    @4324swer Před rokem

    so che è vero ma non saprei dimostrare come dimostrarlo

  • @francescoabbruzzese170

    n^2 = -1 mod 3 n^2 = 2 mod 3. Ma non esiste nessun numero che elevato al quadrato da 2 mod 3, poichè 1^2 = 1 mod 3 e 2^2 = 1 mod 3.

    • @GaetanoDiCaprio
      @GaetanoDiCaprio  Před rokem

      Sì, manca 0^2 e la dimostrazione è completa (la seconda del video)

    • @francescoabbruzzese170
      @francescoabbruzzese170 Před rokem

      @@GaetanoDiCaprio il caso 0 è banale, indipendentemente dal mod rimane sempre 0. Si la dimostrazione usa le congruenze come la tua seconda, ma secondo me mette in luce una tecnica dimostrativa migliore: semplificare ed usare al massimo i vincoli del problema prima di passare all analisi di tutte le possibilità, in modo da semplificare e ridurre i gli scenari da analizzare. In questo caso non si guadagna molto, ma in caso anche un poco piú complessi passare ad analizzare subito tutti gli scenari possibili diventa quasi impossibile.

    • @GaetanoDiCaprio
      @GaetanoDiCaprio  Před rokem

      @@francescoabbruzzese170 In teoria sono d'accordo, ma in concreto i casi "banali" in matematica vanno comunque citati altrimenti la dimostrazione è semplicemente incompleta. Comunque sono molto curioso di eventuali altre tue proposte magari su video che propongono problemi più complessi. Ad esempio, mi piacerebbe sapere il tuo punto di vista sul mio video dal titolo "pochi trovano tutte le soluzioni". Grazie!

  • @Macisordi
    @Macisordi Před rokem

    Un dettaglio, è più facile forse scrivere -1 al posto di 2 così si capisce al volo

    • @GaetanoDiCaprio
      @GaetanoDiCaprio  Před rokem

      A cosa ti riferisci? Scrivere -1 al posto di 2 dove? In che passaggio?

    • @Macisordi
      @Macisordi Před rokem

      @@GaetanoDiCaprio quando si usano le congruenze, in base 3. Per comodità io uso -1, 0, 1 perché è più facile per alcune operazioni (vedi i quadrati)

    • @GaetanoDiCaprio
      @GaetanoDiCaprio  Před rokem

      @@Macisordi più "comodo" non vuol dire più "chiaro", ossia che si capisce al volo. In termini di resti, è più corretto parlare di resto 0, 1, 2. Grazie comunque del commento

    • @Macisordi
      @Macisordi Před rokem

      @@GaetanoDiCaprio Sarebbe come ostinarsi ad usare le 12 ore perché sull’orologio è solo scritto da 1 a 12.

  • @ric52
    @ric52 Před rokem

    magari sto dicendo una fesseria grossa come una casa però la dico lo stesso: mi viene da cercare le intersezioni tra y= x^2 +1 e y = x/3 se non ce ne sono è fatta... se ce ne sono bisogna vedere se sono intere
    si prega di non insultare... sono un medico permaloso ...😂 e sono 50 anni che sono uscito dal liceo 😭

    • @GaetanoDiCaprio
      @GaetanoDiCaprio  Před rokem

      L'insulto non appartiene al mio vocabolario, non si preoccupi! Mi dispiace ma l'idea è errata: quell'intersezione (che non esiste) corrisponderebbe semplicemente a un numero che elevato al quadrato e aumentato di 1 fosse uguale alla sua terza parte.

    • @ric52
      @ric52 Před rokem

      si, certo ... è certamente vero quello che afferma ma avevo l'idea che le due equazioni potessero rappresentare una l'insieme dei numeri soddisfacenti la condizione 1 e la seconda l'insieme dei numeri divisibili per 3 ma non è così... vabbè sto invecchiando...e pure male.. scusate se "ho disturbato"

  • @Maxito9403
    @Maxito9403 Před rokem

    Semplicemente dividerei per casi:
    n=0 mod3 → n²=0 mod3 → n²+1=1 mod3, dunque n²+1 non è divisibile per 3
    n=1 mod3 → n²=1 mod3 → n²+1=2 mod3, dunque anche in questo caso n non è divisibile per 3
    n=2 mod3 → n²=1 mod3 → n²+1=2 mod3, e anche in questo caso n non è divisibile per 3.
    Poiché ogni numero intero è congruo a 0, 1 o 2 modulo 3, nessun successivo di un quadrato è divisibile per 3
    *Il segno "=" è utilizzato impropriamente perché non trovo il simbolo di congruenza nella tastiera :')

    • @GaetanoDiCaprio
      @GaetanoDiCaprio  Před rokem +1

      Hai poi visto il video? Identica alla dimostrazione 2!

  • @josemanuico5613
    @josemanuico5613 Před rokem

    False. 7x7+1=50

  • @claudiobonanno2620
    @claudiobonanno2620 Před rokem

    non è divisibile per tre

  • @silvanodimaula6305
    @silvanodimaula6305 Před rokem

    No, non è divisibile, è logico.

    • @GaetanoDiCaprio
      @GaetanoDiCaprio  Před rokem

      "logico" non vuol dire "ovvio"

    • @silvanodimaula6305
      @silvanodimaula6305 Před rokem

      @@GaetanoDiCaprio non vuol dire ovvio, ma non potrà mai essere divisibile.

    • @GaetanoDiCaprio
      @GaetanoDiCaprio  Před rokem +2

      @@silvanodimaula6305 in matematica ciò che non è ovvio va dimostrato, dire "è logico" non è rilevante da un punto di vista matematico

  • @preparedgalaxy872
    @preparedgalaxy872 Před rokem

    Mi sentivo fico a dire "basta mettere n=1,414".
    Ho letto dopo che si trattava di numeri INTERI ahahaha

  • @redheart5216
    @redheart5216 Před rokem

    Grazie